28
$\begingroup$

Without using a calculator or a computer can you determine which of these two numbers is bigger: 2.253.375 or 3.3752.25?

$\endgroup$

2 Answers 2

26
$\begingroup$

They are

actually exactly the same number.

  • You need to write each base as a power of the same number. $ ??2.25=\frac94=\Bigl(\frac32\Bigr)^{\!2} ??\qquad ??3.375=\frac{27}{8}=\Bigl(\frac32\Bigr)^{\!3} ??$

  • Then Substitute those expressions:

$$ \begin{aligned} 2.25^{3.375} &=\Bigl(\bigl(\tfrac32\bigr)^{2}\Bigr)^{\;(\tfrac32)^{3}} =\Bigl(\tfrac32\Bigr)^{\,2\;(\tfrac32)^{3}}\\ 3.375^{2.25} &=\Bigl(\bigl(\tfrac32\bigr)^{3}\Bigr)^{\;(\tfrac32)^{2}} =\Bigl(\tfrac32\Bigr)^{\,3\;(\tfrac32)^{2}} \end{aligned} $$

  • Lastly compare the exponents of $\tfrac32$:

$$ 2\bigl(\tfrac32\bigr)^{3}=2\cdot\frac{27}{8}=\frac{27}{4}=6.75 \qquad 3\bigl(\tfrac32\bigr)^{2}=3\cdot\frac{9}{4}=\frac{27}{4}=6.75 $$

Since both powers reduce to $\bigl(\tfrac32\bigr)^{6.75}$, the two original numbers

are equal.

Equivalently,

raising each to the 4th power gives $\frac{3^{27}}{2^{27}}$ in both cases.


Additional Math

Key facts about $a^{\,b}=b^{\,a}$

step pick $n$ choose $a_n=(1+\tfrac1n)^{n}$ and $b_n=(1+\tfrac1n)^{n+1}$ result
1 $n=1$ $a_1=2$ $b_1=4$ $2^{4}=4^{2}=16$
2 $n=2$ $a_2=2.25$ $b_2=3.375$ $2.25^{3.375}=3.375^{2.25}$
? any $n\ge1$ $a_n$ lies below e $b_n=a_n(1+1/n)$ lies above e still $a_n^{\,b_n}=b_n^{\,a_n}$

Why it works

  1. From $a^{\,b}=b^{\,a}$ we get $\dfrac{\ln a}{a}=\dfrac{\ln b}{b}$. Picking consecutive points on each side of the maximum of $f(x)=\dfrac{\ln x}{x}$ (which occurs at $x=e$) guarantees equality.

  2. The family above arises by taking those two points to be in the fixed ratio $ \dfrac{b_n}{a_n}=1+\dfrac1n$.

  3. Limit as $n\to\infty$:

    $$ a_n=(1+\tfrac1n)^{n}\;\longrightarrow\;e \qquad b_n=a_n\Bigl(1+\tfrac1n\Bigr)\;\longrightarrow\;e $$

    because $(1+1/n)^{n}$ is the classic defining sequence for $e$ and the extra factor $(1+1/n)$ tends to 1. Thus both sequences “pinch’’ toward Euler’s number while preserving the identity $a_n^{\,b_n}=b_n^{\,a_n}$

enter image description here

$\endgroup$
4
  • 1
    $\begingroup$ Well done! That was incredibly fast. $\endgroup$ Commented 2 days ago
  • $\begingroup$ @DmitryKamenetsky thanks to Sunday chilling :) $\endgroup$
    – Oray
    Commented 2 days ago
  • 1
    $\begingroup$ Thanks for the extra details. $\endgroup$ Commented 2 days ago
  • $\begingroup$ Also a +1 from me. I did make a simplification in your last spoiler, identifying the common value as a 4th root of 3^(27)/2^(27) instead of an eighth root of 3^(54)/2^(54). $\endgroup$ Commented yesterday
4
$\begingroup$

This is the second equality in a sequence going back to Euler (the first one is the familiar $2^4=4^2$).

Let

$x=\left(\dfrac{n+1}{n}\right)^{n}$

$y=\left(\dfrac{n+1}{n}\right)^{n+1}$

Then

$x^y=\left(\dfrac{n+1}{n}\right)^{n×\left(\dfrac{n+1}{n}\right)^{n+1}}$ $=\left(\dfrac{n+1}{n}\right)^{\left(\dfrac{(n+1)^{n+1}}{n^n}\right)}$

and

$y^x=\left(\dfrac{n+1}{n}\right)^{(n+1)×\left(\dfrac{n+1}{n}\right)^{n}}$ $=\left(\dfrac{n+1}{n}\right)^{\left(\dfrac{(n+1)^{n+1}}{n^n}\right)}$

Thereby $x^y=y^x$. Here

$x=2.25=9/4=(3/2)^2, y=3.375=27/8=(3/2)^3$

which matches the Euler form with $n=2$. The next member of the sequence, with $n=3$, would be

$(64/27)^{256/81}=(256/81)^{64/27}$

Euler also showed that the formula above contains all the positive rational solutions for $x^y=y^x$ apart from $y=x$.

The values of $ x^y=y^x$ with this formula decrease with increasing $n$. Thus $16=2^4=4^2$ is greater than the limiting value of $e^e(\approx 15.154)$, with $2.25^{3.375}$ and other values in the sequence falling in-between.

$\endgroup$
3
  • 1
    $\begingroup$ More generally, any real $R \in (0,1)$ determines a pair $(R^\frac{1}{R-1},R^\frac{R}{R-1})$ satisfying $x^y=y^x$. $\endgroup$ Commented 2 days ago
  • $\begingroup$ Yes, but the parameter used here must be an integer for a rational solution. As mentioned, proof goes back to Euler. $\endgroup$ Commented yesterday
  • $\begingroup$ Any positive real $R$ will work, although $R$ and $1/R$ lead to the same pair in opposite orders. $\endgroup$ Commented yesterday

Your Answer

By clicking “Post Your Answer”, you agree to our terms of service and acknowledge you have read our privacy policy.

Start asking to get answers

Find the answer to your question by asking.

Ask question

Explore related questions

See similar questions with these tags.